You are on page 1of 1

Art of Problem Solving

SIGN IN REGISTER

Summer schedule is now available! Enroll today to


×
secure your spot!

CYCLIC INEQUALITY WITH FRACTIONS AND ROOTS

High School Olympiads V


Cyclic inequality with fractions and roots J
N Reply G H
Source: Baltic Way 2020, Problem 2

Tintarn K
#1 Nov 14, 2020, 11:55 PM • 1 Y
Y
Let be positive real numbers such that
. Prove that

pi_quadrat_sechstel K
#2 Nov 15, 2020, 12:22 AM • 4 Y
Y
My solution during the contest:

There are with


. We need to prove

Define by

Note that . We need to prove

Since we have

Since equality holds if and only if .


We have analogous inequalities for . Thus

Equality can only hold for


which contradicts .

pi_quadrat_sechstel K
#3 Nov 15, 2020, 4:50 AM
Y
If , we have and

Thus can not be replaced by a better constant.

sqing K
#4 Nov 15, 2020, 11:43 AM • 1 Y
Y
Let be positive real numbers such that
. Prove that

*
Romania Team Selection Test 1993:
Find the greatest real numbers such that

is true for all positve reals numberes .


For and positive reals, prove that

This post has been edited 2 times. Last edited by sqing, Aug 6, 2022, 11:01
AM

Deligne K
#5 Dec 3, 2020, 6:18 PM
Y
Since there exist such that
and . After making this

substitution, the original inequality becomes

Observe that by AM-GM

.
Therefore

and hence

It is clear that the equality can not hold, hence


the conclusion.
This post has been edited 3 times. Last edited by Deligne, Dec 4, 2020, 5:08
AM
Reason: corrected a mistake

quirtt K
#6 Dec 3, 2020, 8:24 PM
Y
Deligne wrote:

Observe that by AM-GM

How did you add 1 and got that in the RHS?

Math00954 K
#7 Dec 3, 2020, 8:33 PM
Y
Directly AM-GM on .

MrOreoJuice K
#8 Dec 3, 2020, 8:48 PM • 1 Y
Y
We know that so let's do some
substitutions along with expanding the whole
thing

L1
Now Enough to show that the above result is true
Applying AM-GM we get

note

Now

(raising to the power 6 both sides)


now since
we
could divide both sides by

which is true , hence our required result is true.


Now L1

L1
Hence proved
This post has been edited 1 time. Last edited by MrOreoJuice, Dec 15,
2020, 6:20 PM

Deligne K
#9 Dec 4, 2020, 4:12 AM
Y
quirtt wrote:

Deligne wrote:

Observe that by AM-GM

How did you add 1 and got that in the


RHS?

You are right, I did mess up there. Let me try


again.

By AM-GM we have

and therefore

Hence

and again the equality cannot hold.

snakeaid K
#10 Dec 12, 2020, 12:51 AM
Y
Substitute , , . Then the

inequality is

Let , , . Then the inequality


is

Which is true since

The equality when applying AM-GM cannot hold


since then , , ,
which is surely not true.

IMD2 K
#11 Sep 22, 2021, 4:08 PM
Y
Substitute same as above to obtain equivalence
to:

Homogenize with , together


with substitution , ,
to obtain that the inequality is
equivalent to:

Under the condition


. Note that
because ,
and the result follows. We can't obtain equality in
all 3 cases, cause that would imply that
, which would mean that the
condition doesn't hold.

sqing K
#12 Nov 16, 2021, 10:02 AM
Y
Let be positive real numbers such that
. Prove that

jasperE3 K
#13 Nov 16, 2021, 11:42 AM • 1 Y
Y

Tintarn wrote:

Let be positive real numbers such


that . Prove that

Let . We need to

show:

This inequality is homogenous in . WLOG


, then consider the function

We need to show . To
that end, claim that for .
Indeed:

for any such , with equality iff . Using


this:

with equality iff , which


contradicts that . So the
inequality is strict, and we're done.
This post has been edited 1 time. Last edited by jasperE3, Nov 16, 2021,
3:27 PM

pi_quadrat_sechstel K
#14 Nov 16, 2021, 3:05 PM
Y
jasperE3 wrote:

Tintarn wrote:

Let be positive real


numbers such that .
Prove that

...

for any such , with equality iff .


...

The euality case is .

N Quick Reply

© 2024 Art of Problem Solving


About Us • Contact Us • Terms • Privacy

You might also like